Difference between revisions of "2009 AMC 12A Problems/Problem 25"

(Solution)
(Solution 2=)
Line 31: Line 31:
 
Our answer is <math>|a_{2009}| = \boxed{\textbf{(A)}\ 0}</math>.
 
Our answer is <math>|a_{2009}| = \boxed{\textbf{(A)}\ 0}</math>.
  
=====Solution 2======
+
== Solution 2 ==
 
First note how this sequence can be rewritten as tans, and each term is tan of sum of previous 2 angles. Thus we can ignore the tan for now and just focus on the angle itself; first few are 45, 30, 75, 105, 180, and since the question is asking for absolute value of the 2009th term, 180°=0°. This means after the first 180° each successive odd term mod 180 is 0 and each even term is 105 mod 180. Since 2009 is odd, you find tan(0) which is just 0.
 
First note how this sequence can be rewritten as tans, and each term is tan of sum of previous 2 angles. Thus we can ignore the tan for now and just focus on the angle itself; first few are 45, 30, 75, 105, 180, and since the question is asking for absolute value of the 2009th term, 180°=0°. This means after the first 180° each successive odd term mod 180 is 0 and each even term is 105 mod 180. Since 2009 is odd, you find tan(0) which is just 0.
  

Revision as of 13:52, 11 January 2017

Problem

The first two terms of a sequence are $a_1 = 1$ and $a_2 = \frac {1}{\sqrt3}$. For $n\ge1$,

\[a_{n + 2} = \frac {a_n + a_{n + 1}}{1 - a_na_{n + 1}}.\]

What is $|a_{2009}|$?

$\textbf{(A)}\ 0\qquad \textbf{(B)}\ 2 - \sqrt3\qquad \textbf{(C)}\ \frac {1}{\sqrt3}\qquad \textbf{(D)}\ 1\qquad \textbf{(E)}\ 2 + \sqrt3$

Solution 1

Consider another sequence $\{\theta_1, \theta_2, \theta_3...\}$ such that $a_n = \tan{\theta_n}$, and $0 \leq \theta_n < 180$.

The given recurrence becomes

\begin{align*} a_{n + 2} & = \frac {a_n + a_{n + 1}}{1 - a_na_{n + 1}} \\ \tan{\theta_{n + 2}} & = \frac {\tan{\theta_n} + \tan{\theta_{n + 1}}}{1 - \tan{\theta_n}\tan{\theta_{n + 1}}} \\ \tan{\theta_{n + 2}} & = \tan(\theta_{n + 1} + \theta_n) \end{align*}

It follows that $\theta_{n + 2} \equiv \theta_{n + 1} + \theta_{n} \pmod{180}$. Since $\theta_1 = 45, \theta_2 = 30$, all terms in the sequence $\{\theta_1, \theta_2, \theta_3...\}$ will be a multiple of $15$.

Now consider another sequence $\{b_1, b_2, b_3...\}$ such that $b_n = \theta_n/15$, and $0 \leq b_n < 12$. The sequence $b_n$ satisfies $b_1 = 3, b_2 = 2, b_{n + 2} \equiv b_{n + 1} + b_n \pmod{12}$.

As the number of possible consecutive two terms is finite, we know that the sequence $b_n$ is periodic. Write out the first few terms of the sequence until it starts to repeat.

$\{b_n\} = \{3,2,5,7,0,7,7,2,9,11,8,7,3,10,1,11,0,11,11,10,9,7,4,11,3,2,5,7,...\}$

Note that $b_{25} = b_1 = 3$ and $b_{26} = b_2 = 2$. Thus $\{b_n\}$ has a period of $24$: $b_{n + 24} = b_n$.

It follows that $b_{2009} = b_{17} = 0$ and $\theta_{2009} = 15 b_{2009} = 0$. Thus $a_{2009} = \tan{\theta_{2009}} = \tan{0} = 0.$

Our answer is $|a_{2009}| = \boxed{\textbf{(A)}\ 0}$.

Solution 2

First note how this sequence can be rewritten as tans, and each term is tan of sum of previous 2 angles. Thus we can ignore the tan for now and just focus on the angle itself; first few are 45, 30, 75, 105, 180, and since the question is asking for absolute value of the 2009th term, 180°=0°. This means after the first 180° each successive odd term mod 180 is 0 and each even term is 105 mod 180. Since 2009 is odd, you find tan(0) which is just 0.

Note

It is not actually difficult to list out the terms until it repeats. You will find that the period is 7 starting from term 2.

See also

2009 AMC 12A (ProblemsAnswer KeyResources)
Preceded by
Problem 24
Followed by
Last question
1 2 3 4 5 6 7 8 9 10 11 12 13 14 15 16 17 18 19 20 21 22 23 24 25
All AMC 12 Problems and Solutions

The problems on this page are copyrighted by the Mathematical Association of America's American Mathematics Competitions. AMC logo.png